Difference between revisions of "1988 AJHSME Problems/Problem 7"

m
Line 13: Line 13:
 
{{AJHSME box|year=1988|num-b=6|num-a=8}}
 
{{AJHSME box|year=1988|num-b=6|num-a=8}}
 
[[Category:Introductory Algebra Problems]]
 
[[Category:Introductory Algebra Problems]]
 +
{{MAA Notice}}

Revision as of 23:55, 4 July 2013

Problem

$2.46\times 8.163\times (5.17+4.829)$ is closest to

$\text{(A)}\ 100 \qquad \text{(B)}\ 200 \qquad \text{(C)}\ 300 \qquad \text{(D)}\ 400 \qquad \text{(E)}\ 500$

Solution

We estimate the first thing to be $2.5$, the second thing to be $8$, and the third thing to be $10$. We now have $2.5\cdot 8\cdot 10=25\cdot 8=200\Rightarrow \mathrm{(B)}$.

See Also

1988 AJHSME (ProblemsAnswer KeyResources)
Preceded by
Problem 6
Followed by
Problem 8
1 2 3 4 5 6 7 8 9 10 11 12 13 14 15 16 17 18 19 20 21 22 23 24 25
All AJHSME/AMC 8 Problems and Solutions

The problems on this page are copyrighted by the Mathematical Association of America's American Mathematics Competitions. AMC logo.png